0
$\begingroup$

Is there any way to solve an linear optimization problem with quadratic integer constraint?

E.g. $\max a^Tx$, $x=\langle x_1,x_2,\cdots,x_n \rangle$

s.t. $x_ix_j<b_{ij}$, $\forall x_i,x_j \in x$

$x_i \in \{0,1\}, \forall x_i \in x$

$\endgroup$

1 Answer 1

2
$\begingroup$

Since $x$ is binary and the product only can be $0$ or $1$, your model is a bit odd with the strict inequality. If $b_{ij} >1$ then $x_i$ and $x_j$ are arbitrary, and if $b_{ij} \leq 1$ then you must model the fact that one of the terms has to be zero, which you can do by $x_i + x_j \leq 1$.

$\endgroup$
4
  • $\begingroup$ I see what you mean… I also realized my model looks weird with the 0 or 1 value for $x_i$… But if $b_{ij}=1$ then $x_i$ and $x_j$ can be 1 at the same time $\endgroup$ Commented May 6, 2018 at 16:05
  • $\begingroup$ For $b_{ij}=1$, $x_i$ and $x_j$ can also be arbitrary. So is there any way to mathematically specify the constraint to $b_{ij}<1$? $\endgroup$ Commented May 6, 2018 at 16:12
  • 1
    $\begingroup$ No, for $b_{ij}=1$ one of them has to be zero, since the product has to be $0$ in that case $\endgroup$ Commented May 6, 2018 at 18:12
  • 1
    $\begingroup$ ...for $0 < b_{ij} < 1$ the product has to be $0$. For $b\leq 0$ the problem is infeasible. You seem to fail to appreciate that you have a strict inequality (which I commented upon in my answer) $\endgroup$ Commented May 6, 2018 at 18:15

You must log in to answer this question.

Start asking to get answers

Find the answer to your question by asking.

Ask question

Explore related questions

See similar questions with these tags.